Quick viewing(Text Mode)

CHAPTER 128 Bone and Joint Infections

CHAPTER 128 Bone and Joint Infections

CHAPTER 128 and Joint Infections

Neha P. Raukar | Brian J. Zink

PRINCIPLES examinations and helps guide management, including antibiotic therapy and surgical intervention. Background Anatomy and Physiology Historically, bone and joint infections (BJIs) have been described in grim terms. Aids to Surgery, written in 1919, noted that “acute Histologically, bone tissue is classified as compact or spongy. infective … is a very fatal disease.” With septic Compact bone forms the shaft of long and outer shell of arthritis, “the patient becomes exhausted from toxaemia or all bone. Spongy bone is found at the ends of long bones and pyemia,” and “ankylosis is the usual most favourable termina- makes up irregular bones. Compact bone is dense and without tion.”1 Advances in diagnostic methods, antibiotic therapy, and cavities and consists of longitudinally running Haversian systems, surgical techniques have resulted in better patient outcomes; which contain Haversian canals that house vasculature and nerves. however, new challenges are arising. Antibiotic resistance is evolv- Spongy bone, conversely, consists of a bony lattice, the trabeculae, ing, and many patient subsets have reduced host immunity. This which contains marrow, is more metabolically active, and is less combination results in greater complexity in the management of dense than compact bone. The central Haversian canals run paral- BJIs than has ever been encountered. The emphasis of modern lel to the long axis of the bone and contain the blood supply and management of BJIs has shifted from prevention of sepsis and reticular connective tissue for the Haversian system. Spongy bone, death to prompt diagnosis, initiation of treatment, and avoidance also called cancellous or medullary bone, has numerous cavities, of the complications and morbidity associated with chronic bone is located within the medullary cavity, and consists of extensively or joint infections. connected trabeculae. The overall occurrence of BJIs appears to have remained The gross structure of long bones can be divided into several constant during the past 4 decades.2 In hospitalized patients in the sections. The diaphysis is the shaft of the bone and contains the United States, the incidence is approximately 1%. Osteomyelitis compact cortical bone with an overlying periosteum and a medul- in children younger than 13 years occurs in 1 in 5000, whereas the lary canal containing marrow. The metaphysis is the junctional incidence of septic arthritis ranges from 5.5 to 12/100,000 indi- region between the epiphysis and diaphysis. The metaphysis viduals.2 In contrast to the rest of the world, there is no correlation contains abundant trabecular bone, but the cortical bone thins between socioeconomic factors or race and the incidence of BJI here relative to the diaphysis. Finally, the epiphysis is the area at in the United States. Both bone and joint infections show a either end of a long bone and is made up of abundant trabecular bimodal age distribution, occurring most commonly in people bone and a thin shell of cortical bone (Fig. 128.1). In the skeletally younger than 20 years or older than 50 years. In children, BJIs mature individual, the epiphysis of most bones is involved in usually occur in previously healthy individuals, with boys having articulation and, instead of being covered by a periosteum, is a slightly increased susceptibility to bone infections. In adults, covered with a thin layer of articulating cartilage, a very thin layer there are several known risk factors that lead to a higher risk of secretory cells sitting on a loose fibrous stroma that allows of BJIs. frictionless movement of the bones. Orthopedic infections can be classified according to the site of Joints are enclosed by a synovial capsule, which consists of a involvement and include osseous (osteomyelitis), articular (septic, dense fibrous connective tissue that offers structural integrity and pyogenic, or suppurative arthritis), bursal (septic bursitis), subcu- is lined with synovial cells that secrete synovial fluid. This forms taneous (cellulitis or abscess), muscular (infectious myositis or a sleeve around the articulating bones to which it is attached. In abscess), and tendinous (infectious tendinitis or tenosynovitis) some joints, such as the shoulder, hip, and knee, the synovial varieties. The terms osteomyelitis literally means of membrane extends beyond the epiphysis and attaches to the the marrow of the bone, but it is colloquially used to refer to metaphysis. This anatomic relationship allows bacteria to spread infection in any part of the bone. directly from the metaphysis into the joint. Infectious processes can also be categorized by their onset and are generally designated as acute, subacute, or chronic. An acute Pathophysiology infection is one that lasts less than 2 weeks, a subacute infection is one that lasts 2 to 6 weeks, and chronic infections are those that Osteomyelitis is an infection of the bone and medullary cavity. last longer than 6 weeks. Chronic osteomyelitis is also used to Bone is typically resistant to infection unless it is subjected to define a bone infection that fails to respond to a a normal course trauma, disruption of blood flow that deprives the bone of normal of antibiotic therapy. A histologic diagnosis of chronic osteomy- host immunity, a large inoculum of blood-borne or external elitis depends on the presence of necrotic bone. microorganisms, or a foreign body. Hematogenous inoculation For the emergency clinician, the most practical way to classify usually starts in the metaphysis, given the slow flow of blood in osteomyelitis is as hematogenous, which is more common, or the sinusoidal blood vessels. Acute inflammatory cells migrate to contiguous, with the contiguous type further subdivided based on the area, causing edema, vascular congestion, and small vessel the presence or absence of vascular insufficiency. This method of thrombosis, which then leads to an increase in the intraosseous classification assists in the interpretation of diagnostic imaging pressure compromising blood flow to the bone. Eventually, lack

1693 Downloaded for dr.Rahmat Dani Satria, M.Sc, Sp.PK ([email protected]) at Universitas Gadjah Mada from ClinicalKey.com by Elsevier on July 28, 2018. For personal use only. No other uses without permission. Copyright ©2018. Elsevier Inc. All rights reserved. 1694 PART III Medicine and Surgery | SECTION Twelve Infectious Diseases

Compact bone Epiphysis Spongy bone Haversian canals

Trabeculae Enlarged below

Haversian Diaphysis canals

Volkmann’s canals

Lamellae

Metaphysis Marrow Epiphysis or medullary A B cavity

Lacunae

Haversian canal

Canaliculi

C

Fig. 128.1. Schematic drawing of long bone. A, Regions of long bone. B, Cross-sectional structure of long bone. C, Microscopic structure.

of blood supply to the medullary canal and periosteum leads to are at varying stages of metabolism—some are active, some are areas of necrotic bone termed sequestra. Bony tissue attempts to slow-growing, and some are dormant. Antibiotics target meta- compensate for the tensile stresses caused by infection by creating bolically active bacteria, such as those in the single cell state new bone around the areas of necrosis. This new bone deposition (planktonic state), but bacteria in other stages in the biofilm com- is called an . Given that there is significantly reduced munity are more resistant to the effects of antibiotics. Further- blood supply to this necrotic bone tissue, bacterial infection is more, Gram staining only identifies planktonic bacteria, which often difficult to eradicate with medication alone and, frequently, helps explain why Gram stains of aspirated synovial fluid in a chronic osteomyelitis requires a combination of surgical débride- suspected septic joint are often negative; therefore, a definitive ment and antibiotic therapy. diagnosis is made only by culture of the synovial fluid aspirate or The evolution of blood flow patterns at the metaphyseal- synovial tissue. Biofilm formation also explains why optimal treat- epiphyseal junction and development of vascular anatomy explain ment of a septic joint, especially of prosthetic joints, involves the pathologic features of hematogenous osteomyelitis in the complete surgical débridement. different age groups. In neonates and infants, osteomyelitis readily Hematogenous spread of bacteria causes almost all cases of advances from the metaphysis to the epiphysis and adjacent joint osteomyelitis in children and in the subset of adults who have space, leading to septic arthritis. After the first year of life, the . In the appendicular skeleton of adults, infection usually spreads laterally through Volkmann’s canals, such as in the foot, hand, skull, maxilla, and mandible, osteomy- breaks through the cortex, and lifts the periosteum to form a elitis usually occurs by spread of the pathogens from a contiguous subperiosteal abscess. In the adult, after the epiphyseal plate source of infection or direct implantation. Head and neck ossifies, anastomoses form between the metaphyseal and epiphy- osteomyelitis is usually caused by sinus disease and odontogenic seal blood vessels and infection can once again spread from the infection. metaphysis to the epiphysis and eventually into the synovium Infections from direct implantation of bacteria are caused by and joint space. In addition, the periosteum becomes firmly deep puncture wounds, such as by an animal bite, and tend attached to the underlying bone, limiting subperiosteal abscess to occur in the hands and feet. Although cats account for only formation. 10% of animal bites, significant infection results from 20% to Bacteria congregate in a highly structured community, the 50% of cat bites versus only 5% of dog bites because of the biofilm, which plays an important role in the pathogenesis of morphology of feline teeth. Most human bite injuries are related septic arthritis and osteomyelitis. Within the biofilm, the bacteria to fistfights and contamination of the metacarpophalangeal joints

Downloaded for dr.Rahmat Dani Satria, M.Sc, Sp.PK ([email protected]) at Universitas Gadjah Mada from ClinicalKey.com by Elsevier on July 28, 2018. For personal use only. No other uses without permission. Copyright ©2018. Elsevier Inc. All rights reserved. CHAPTER 128 Bone and Joint Infections 1695 and metacarpals, with infection due to oral flora. Direct implanta- susceptible to BJIs, especially those who have implanted prosthetic tion of pathogens is also common with open fractures and surgical devices. instrumentation. Although most BJIs are bacterial, other pathogens include Septic arthritis is usually a consequence of hematogenous viruses, fungi, and parasites. The microbiology of osteomyelitis spread unless there is direct injection of bacteria into the joint. and septic arthritis is a function of several host and environmental The lack of a basement membrane makes the highly vascular factors. A patient’s living environment also has some role in synovium vulnerable to bacterial seeding. Infection occurs first in determining the incidence of BJIs. For example, people living in the synovium, spreads into joint fluid, and finally affects the crowded conditions where tuberculosis is prevalent are at increased articular cartilage. Bacterial enzymes and toxins directly damage risk for tubercular BJIs, whereas older patients in hospitals and cartilage. The synovial membrane responds to infection by institutions may be more susceptible to infections with gram- increasing synovial fluid production, resulting in a large joint negative bacteria. A summary of the organisms that cause osteo- effusion, and progresses to ischemic damage to cartilage. Even a and septic arthritis is presented in Tables 128.1 and 128.2. small bacterial load in the joint space elicits profound and persis- The following points deserve special mention: tent inflammatory and immune responses. Bacteria can be cleared • le In al ag groups except neonates, S. aureus is the leading cause from the joint, resulting in a sterile-appearing inflammatory of osteomyelitis. In neonates, group B streptococci, Escherichia response.3 In response to infection, synovial cells and polymor- coli and other gram-negative coliforms, and Staphylococcus phonuclear leukocytes release lysosomal enzymes, which irrevers- epidermidis are the most common pathogens responsible for ibly degrade articular cartilage, creating a painful joint with BJIs. limited range of motion. In addition, other structures that are • Since the introduction of the vaccine, Haemophilus influenzae enclosed within or adjacent to the synovium, such as bursae, type b, once a common cause of septic arthritis and osteomy- tendons, and bone, may also become damaged in those with septic elitis in children younger than 2 years, has essentially disap- arthritis.3 peared as a pathogen in vaccinated children.4 Another gram-negative coccobacillus in the Neisseriaceae family, Kin- Causes and Microbiology gella kingae has been encountered with increasing frequency. K. kingae can be part of the normal flora of the nasopharynx; Typically, hematogenous osteomyelitis or septic arthritis is caused like H. influenzae, it can be spread hematogenously to bones by a single strain of bacterium, with gram-positive organisms and joints. It is a fastidious organism and may be mistaken for being responsible for most infections. Even though gram-negative Haemophilus or Neisseria spp. organisms account for 43% of cases of community-acquired • P. aeruginosa has been reported as a cause of cervical spine bacteremia, they result in only about 10% of septic arthritis cases. osteomyelitis in injection drug users and lumbar spine osteo- Trauma predisposes patients to osteomyelitis by environmental myelitis in patients with urinary catheters in place for a long pathogens. Patients who are wounded or sustain open fractures time. Pseudomonas colonizes the rubber and plastic inserts in in fresh water are susceptible to infections with the gram-negative footwear and is therefore seen in soft tissue infections and bacillus Aeromonas hydrophila. People who are bitten by animals, osteomyelitis of the foot after a puncture wound. particularly dogs and cats, are at risk for the development of • In older adults and patients with diabetes, gram-negative osteomyelitis from Pasteurella multocida. Osteomyelitis caused bacteria account for a higher percentage of cases of bone and from human bites is most common in the hand and involves joint infections. human oral flora, such as Streptococcus anginosus, Fusobacterium • Methicillin-resistant S. aureus (MRSA), methicillin-resistant S. nucleatum, and Eikenella spp. In the population of injection drug epidermidis, and vancomycin-resistant enterococci (VRE) have users, Staphylococcus aureus is the most likely cause of infection, emerged as a significant microbiologic problem in the past 2 followed by Pseudomonas spp. Pseudomonas aeruginosa is also an decades. Multiresistant enterococci pose the greatest potential important cause of osteomyelitis in puncture wounds, postsurgi- danger in that no currently available antibiotic regimen is cal wounds, and patients with sickle cell anemia. reliably bactericidal against such organisms. Certain underlying disease states predispose a patient to BJI. Certain disease processes are more likely to be due to a poly- These conditions include diabetes mellitus, sickle cell disease, microbial infection. This would include diabetic foot osteomyeli- AIDS, alcoholism, injection drug use, chronic corticosteroid use, tis, posttraumatic osteomyelitis, and chronic septic arthritis or preexisting joint disease (especially rheumatoid arthritis), and osteomyelitis. Furthermore, anaerobic bacteria can complicate other immunosuppressed states. Postsurgical patients are also polymicrobial infection and may be present more often than is

TABLE 128.1 Microbiology of Bacterial Septic Arthritis as Related to Age of the Patient YOUNG ADULT ENGAGING IN ORGANISM CHILD (%)a HIGH-RISK SEXUAL BEHAVIOR (%) ADULT (%) OLDER ADULT (%) Staphylococcus aureus 10–20 15–20 60–0 45–65 Streptococcus species 5–10 1–5 15–20 10–15 Gram-negative bacterium 1–5 Rare 10–5 15–35 Haemophilus influenza Rareb Rareb Rareb Rareb Neisseria gonorrhoeae 1–5 60–80 1–5 Rare aAges 6 months to 5 years. bWith widespread immunization. Reprinted with permission from Esterhai JL Jr, Rao N: The epidemiology of musculoskeletal infections. In Cierny G 3rd, McLaren AC, Wongworawat MD, editors: Orthopaedic knowledge update: musculoskeletal infection. Rosemont, IL, 2009, American Academy of Orthopaedic Surgeons.

Downloaded for dr.Rahmat Dani Satria, M.Sc, Sp.PK ([email protected]) at Universitas Gadjah Mada from ClinicalKey.com by Elsevier on July 28, 2018. For personal use only. No other uses without permission. Copyright ©2018. Elsevier Inc. All rights reserved. 1696 PART III Medicine and Surgery | SECTION Twelve Infectious Diseases

TABLE 128.2 Microbiology and Initial (Empirical) Antibiotic Treatment of Bone and Joint Infection OSTEOMYELITIS SEPTIC ARTHRITIS AGE GROUP Common Organisms Antibiotic Regimen Common Organisms Antibiotic Regimen Neonate to <3 mo Staphylococcus aureus Ceph 3 S. aureus PRP + Ceph 3 Group B streptococcus PRP + gentamicin Group B streptococcus Alt—PRP + APAG Enterobacteriaceae Consider vancomycin instead of Enterobacteriaceae Consider vancomycin instead Gram-negative rods PRP for MRSA. of PRP for MRSA. 3 mo–14 yr S. aureus PRP + Ceph 3 S. aureus PRP + Ceph 3 Group A streptococcus Alt: vancomycin + Ceph 3, Group A streptococcus Alt—vancomycin + Ceph 3 Haemophilus influenzae chloramphenicol Streptococcus PRP or Ceph 3 with allergy to pneumoniae penicillin or clindamycin with H. influenzae allergy to penicillin + Ceph 3 14 yr–adult S. aureus PRP S. aureus PRP or Ceph 3 Alt—vancomycin Streptococcal spp. Alt—vancomycin + Ceph 3 or Enterobacteriaceae penicillin + aminoglycoside or Ceph 3 INFECTION SUBSETS Sexually active adolescents Neisseria gonorrhoeaea Ceph 3 or adults with acute Alt—spectinomycin or arthritis penicillin if sensitive Chronic osteomyelitis and S. aureus PRP + FLQ + metronidazole diabetic foot infections Enterobacteriaceae Alt—PRP + Ceph 3 + Anaerobic bacteria clindamycin Infected orthopedic joint Staphylococcus aureus Vancomycin + FLQ S. aureus Vancomycin + FLQ prosthesis Staphylococcus epidermidis Alt—imipenem S. epidermidis Alt—PRP + APAG Pseudomonas aeruginosa P. aeruginosa Sickle cell disease Staphylococcus aureus PRP + Ceph 3 S. aureus PRP + Ceph 3 Salmonella sp. Alt—FLQ Salmonella spp. Alt—FLQ Injection drug abuse Staphylococcus aureus Ceph 3 + aminoglycoside P. aeruginosa PRP + APAG or FLQ Pseudomonas aeruginosa Alt—Ceph 3 S. aureus Alt—vancomycin + FLQ Enterobacteriaceae Enterobacteriaceae Plantar puncture wound Pseudomonas aeruginosa AP Ceph P. aeruginosa AP Ceph Alt—FLQ Alt—FLQ Human or animal bites Eikenella corrodens Penicillin ± AC E. corrodens Penicillin ± AC Pasteurella multocida Alt—Ceph 3, TS P. multocida Alt—Ceph 3, TS aConcurrent treatment of Chlamydia trachomatis infection should be given to patients with suspected N. gonorrhoeae septic arthritis. Alt, Alternative antibiotics; APAG, antipseudomonal aminoglycoside; AP Ceph, antipseudomonal cephalosporin (ceftazidime or cefepime); Ceph 3, third-generation cephalosporin (eg, ceftriaxone, cefotaxime, cefamandole, ceftizoxime, ceftazidime, cefazolin, moxalactam); FLQ, fluoroquinolone;MRSA, methicillin-resistant S. aureus; PRP, penicillinase- resistant penicillin (oxacillin, nafcillin, methicillin, amoxicillin-clavulanate [AC]); TS, trimethoprim-sulfamethoxazole.

commonly recognized. This is because standard culture techniques a ram-negative, rickettsia-like organism that frequently causes may be inadequate to identify them. For example, anaerobic osteolytic bone lesions. bacteria are reportedly discovered in up to 40% of cases of chronic osteomyelitis. Mycobacterium tuberculosis may infect bones and joints, usually OSTEOMYELITIS in the axial skeleton. The two most common forms of skeletal infection are vertebral osteomyelitis (Pott’s disease), in which the Clinical Features spine is affected in 50% of cases, and tubercular arthritis, which History and Physical Examination manifests as a chronic, low-grade inflammatory process that resembles rheumatoid arthritis more than acute septic arthritis. The symptoms and signs of osteomyelitis in adults are predictable, Patients with human immunodeficiency virus (HIV) infection although not always present. Patients with osteomyelitis often and AIDS are predisposed to a variety of common and opportu- present with , more reliably seen in children, and rigors and nistic pathogens. Although S. aureus is still the most likely cause may even appear toxic. Systemic complaints of headache, fatigue, of bone and joint infections in patients with AIDS, fungal and malaise, and anorexia are inconsistently reported and are less other atypical organisms should be considered. One unusual but likely with chronic osteomyelitis. In children with lower extremity particularly characteristic form of osteomyelitis in HIV-positive osteomyelitis, a sudden limp or inability to bear weight, localized patients is bacillary angiomatosis. This infection is caused by warmth, swelling, and erythema may be reported. A careful review

Downloaded for dr.Rahmat Dani Satria, M.Sc, Sp.PK ([email protected]) at Universitas Gadjah Mada from ClinicalKey.com by Elsevier on July 28, 2018. For personal use only. No other uses without permission. Copyright ©2018. Elsevier Inc. All rights reserved. CHAPTER 128 Bone and Joint Infections 1697 of the patient’s past medical history should be performed to History, physical examination, identify risk factors that may predispose to bone infection. ESR suggestion of acute osteomyelitis The physical examination findings of osteomyelitis are fairly specific. The predominant symptom of osteomyelitis is pain over the affected bone. Palpation usually elicits point tenderness over the infected segment. Palpable warmth and soft tissue swelling Plain ϩ with erythema may be present, but these findings are variable. In chronic advanced osteomyelitis, the involucrum or may be palpated, and sinus tracts that fistulize to the skin may be Blood cultures Ϫ noted. A so-called sympathetic effusion in the adjacent joint may Start antibiotics develop in patients with osteomyelitis, even when the joint is not Admit patient infected. CT or MRI ϩ Complications Ϫ In addition to the development of chronic osteomyelitis, compli- cations of acute osteomyelitis include bacteremia and sepsis. Depending on the location of osteomyelitis, local extension of an Clinical evidence Clinical evidence invasive suppurative process can lead to septic arthritis, brain strong weak abscess, meningitis, spinal cord compression, pneumonia, and empyema. In children, osteomyelitis damages the developing skeleton. If infection involves the epiphysis, permanent growth Blood cultures Outpatient evaluation alterations can occur, resulting in a shorter or deformed extremity Start antibiotics with 99mTc-MDP on the affected side. Pathologic fractures may occur through sites Admit patient Three-phase scintigraphy of osteomyelitis.

Fig. 128.2. Algorithm for the use of imaging studies in the emergency department diagnosis of osteomyelitis. CT, Computed tomography; ESR, Clinical Subsets of Osteomyelitis erythrocyte sedimentation rate; MRI, magnetic resonance imaging; 99mTc- Osteomyelitis in Children. Osteomyelitis in children tends MDP, technetium Tc-99m–labeled methylene diphosphonate. to be acute, usually arising from hematogenous seeding of bone, and can often be treated with antibiotics alone. Acute hematoge- nous osteomyelitis (AHO) is seen in children as young as 3 months and as old as 16 years. S. aureus is the most common infecting organism in children of all ages, except neonates (see Tables 128.1 and 128.2). As noted, H. influenzae is no longer a common cause of AHO. AHO has a well-established male preponderance (male-female ratio of 2 : 1 to 3 : 1) and involves long bones approximately 80% of the time. The site of infection is usually the distal metaphysis because of its increased vascularity, but up to 30% of AHO occurs in other parts of the bone. Children with AHO may have fever, chills, vomiting, dehydration, and malaise, but they usually do not appear toxic. Most children have characteristic pain, limited use of the limb, and are point tender. The diagnostic evaluation for AHO is shown in Fig. 128.2. Blood cultures are positive for the bacterial cause of osteomyelitis in 60% of patients with AHO. A positive blood culture and physical examination consistent with osteomyelitis may be sufficient for a diagnosis of AHO to be made. Figs. 128.3 and 128.4 show typical radiographs of AHO. Neonatal osteomyelitis is difficult to diagnose because of minimal systemic findings. Osteomyelitis in the neonate is more Fig. 128.3. Radiograph of chronic osteomyelitis. Diffuse sclerosis of the commonly seen after an abnormal pregnancy or delivery and right hemipelvis includes the ischium and iliac bone, with extension to often accompanies other acute illnesses. Multiple sites of bone the right sacroiliac joint. The right acetabulum demonstrates whittling of the femoral head and neck, with marked loss of bone stock. Abutting involvement are found in approximately 50% of reported cases. the right symphysis pubis is increased cortical lucency suggestive of Because of the unique vascular anatomy of the neonate, septic subchondral cystic changes. (Courtesy Dr. Peter Evangelista, Department arthritis often accompanies osteomyelitis. Osteomyelitis of the of Diagnostic Imaging, Rhode Island Hospital, Brown University, Provi- flat bones, such as the facial bones, is more common among dence, RI.) neonates than any other age group. Group B streptococcus is the leading causative bacterium in neonatal osteomyelitis, but staphylococcal species are still common. As with adults, plain Two less common forms of osteomyelitis can occur in children, radiographs are a good initial test because abnormalities are subacute osteomyelitis and chronic recurrent multifocal osteomy- identified within days of development of neonatal osteomyelitis; elitis (CRMO). Subacute osteomyelitis refers to a form of the they are usually abnormal by the time the disease is suspected. In disease in which clinical symptoms and signs are slow to appear, the presence of a normal radiograph, the next step for the emer- and radiographs show small areas of osteomyelitis, usually in the gency clinician who suspects neonatal osteomyelitis is magnetic metaphysis of long bones. Cultures of blood and bone are negative resonance imaging (MRI) in consultation with the orthopedic more than 50% of the time but usually implicate staphylococcal surgical service. species when they are positive.

Downloaded for dr.Rahmat Dani Satria, M.Sc, Sp.PK ([email protected]) at Universitas Gadjah Mada from ClinicalKey.com by Elsevier on July 28, 2018. For personal use only. No other uses without permission. Copyright ©2018. Elsevier Inc. All rights reserved. 1698 PART III Medicine and Surgery | SECTION Twelve Infectious Diseases

AB C

DEF

Fig. 128.4. Radiographic progression of acute osteomyelitis. A, Soft tissue swelling at the medial and lateral aspects of the ankle, with a moderately sized effusion (August 2, 2006). B, Large ankle effusion with extensive soft tissue swelling. There is complete loss of the tibiotalar joint space and widening of the medial joint space, suggesting . There are lucent areas in the distal tibia and fibula, suggestive of hyperemia, and the talus is diffusely sclerotic (September 11, 2006). C, Increased erosion of the medial aspect of the talar dome, with increased joint effusion (October 12, 2006). D, Talar bone destruction with demineralization involving all the osseous structures. There is also a small joint effusion (January 2, 2007). E, of the talus and destruction of the articular surfaces of the tibia and talus is present, consistent with chronic osteomyelitis. There is diffuse and loose bodies within the joint (April 19, 2007). F, There is continued irregularity of the articular surface of the tibia and collapse of the talus. Loose bodies are still present within the joint, and there is a persistent joint effusion and soft tissue swelling (June 14, 2007). (Courtesy Dr. Thomas Egglin, Department of Diagnostic Imaging, Rhode Island Hospital, Brown University, Providence, RI.)

Like subacute osteomyelitis, CRMO usually affects older chil- older adults include intravenous (IV) access devices, indwelling dren (6–10 years) and adolescents. CRMO is characterized by lines, and asymptomatic urinary infections, whereas in younger small foci of infection at various sites in the skeleton. The disease individuals, risk factors include injection drug abuse. The spine is is defined by multiple episodes of indolent infection. Diagnosis is susceptible to bacterial infection because the venous system sur- made by radiography because culture of the bone sites is almost rounding vertebral bodies is valveless, permitting two-way flow of always negative. This disease may be associated with certain pso- blood, and has transverse and longitudinal anastomoses. Infection riatic subtypes. can readily spread to adjacent vertebral bodies. Vertebral osteo- myelitis usually results from hematogenous seeding, direct inocu- Vertebral Osteomyelitis. Vertebral osteomyelitis usually lation at the time of spinal surgery, or contiguous spread from an afflicts older adults in a manner analogous to that of AHO in adjacent infection. A clear source of bacterial hematogenous children and appears to be increasing in frequency as the popula- seeding with positive blood cultures occurs in approximately 40% tion ages and has more chronic medical diseases. Risk factors in of cases of vertebral osteomyelitis. S. aureus (including MRSA) is

Downloaded for dr.Rahmat Dani Satria, M.Sc, Sp.PK ([email protected]) at Universitas Gadjah Mada from ClinicalKey.com by Elsevier on July 28, 2018. For personal use only. No other uses without permission. Copyright ©2018. Elsevier Inc. All rights reserved. CHAPTER 128 Bone and Joint Infections 1699 the most common offending agent, followed by aerobic gram- avascular disk creates a rich environment for the bacteria to flour- negative rods from urinary or gastrointestinal sources. ish. Because of the vascular anatomy, adult pyogenic diskitis Only 10% of patients with vertebral osteomyelitis appear coexists with vertebral osteomyelitis; in children, however, isolated septic or toxic; most patients present with insidious symptoms, diskitis is more common. The patient complains of back pain leading to delays in diagnosis of up to 4 months. Back pain, seen and may refuse to walk. Although bone scintigraphy may show in roughly 90% of patients, is the most common presenting increased uptake in the disk space, MRI better demonstrates the symptom, and physical examination often reveals tenderness over anatomy of diskitis. CT is used to guide aspiration. Cultures of the spinous process. Neurologic deficits are reported in less than the disk from needle aspiration are reported to be positive for 40% of patients with vertebral osteomyelitis and often coincide bacteria 30% to 60% of the time, usually y for S. aureus. The with a concomitant epidural abscess.5 Up to 60% of patients disease typically resolves with nonoperative treatment. with these abscesses present without fever or . On laboratory testing, the erythrocyte sedimentation rate (ESR) and Posttraumatic Osteomyelitis. Posttraumatic osteomyelitis C-reactive protein (CRP) level are elevated in 98% and 100% of is a form of contiguous focus osteomyelitis that results from open cases, respectively.5 Blood culture specimens should be obtained fractures, burns, bites, puncture wounds, and surgery and invasive before antibiotic treatment is initiated. Rapid diagnosis and treat- procedures. At least 10% of open fractures later develop osteomy- ment of this medical emergency starts with the initiation of elitis, with the tibia being the bone that is usually affected. The empirical antibiotics, immediate imaging, and early orthopedic fracture site may be contaminated directly from the environment involvement. or iatrogenically secondary to emergency procedures or surgery. The imaging strategy for the diagnosis of vertebral osteomy- The intraoperative implantation of prosthetic devices increases elitis starts with plain radiography, which may show disk space the chance of infection. Severe damage to adjacent soft tissues narrowing or destruction of the vertebral endplates or vertebral results in a necrotic nidus of infection that can spread to bone. body. Similar to osteomyelitis in other parts of the body, findings Polymicrobial infection is more common with this type of osteo- on plain radiographs are not seen until at least the second week myelitis. The imaging of posttraumatic osteomyelitis is compli- of vertebral infection. Bone scintigraphy has been largely replaced cated by changes induced by surgery and new bone formation in by MRI in further imaging of suspected vertebral osteomyelitis. the fracture; therefore, the optimal imaging modalities are MRI Although computed tomography (CT) is good for defining bone and CT. destruction and is often used to assist needle aspiration of the Osteomyelitis due to direct inoculation associated with joint lesion, MRI can identify an epidural abscess and rule out other arthroplasty typically becomes evident about 12 weeks after noninfectious vertebral conditions. MRI has a sensitivity of 90% surgery, but these patients generally do not report relief of for vertebral osteomyelitis, with T2-weighted images most valu- their pain after surgery. Patients who have symptoms of infection able in establishing the diagnosis. more than 12 weeks after surgery and who have postoperative Vertebral infections can occur in the lumbar (58%), thoracic improvement of their pain are considered to have a hematogenous (30%), and cervical (11%) spine. Cervical spine osteomyelitis can source of infection. If either of these presentations is recognized cause a retropharyngeal abscess; lumbar spine osteomyelitis may within the first 2 weeks of onset of infectious symptoms, the be complicated by a psoas muscle abscess. The spinal cord may prosthesis is considered salvageable. After 2 weeks, the chance also suffer ischemic injury if the vertebral infection causes septic of eradicating the infection without removal of the prosthesis thrombosis or compression of local blood vessels. When osteomy- decreases substantially. elitis affects the thoracic spine, infection can spread to the chest. The most common form of postsurgical osteomyelitis is infec- Paraspinal abscesses, reactive pleural effusions, and empyema tion of a hip prosthesis, which occurs in 1% to 5% of hip replace- have been reported and may mislead the emergency clinician to ment surgeries. Postsurgical osteomyelitis is difficult to diagnose. suspect that the primary problem is not in the spine. The most Fever is often absent, and the patient often presents with a painful dreaded complication of vertebral osteomyelitis is the spread of joint. S. aureus and S. epidermidis account for 75% of postsurgical infection into the spinal canal, development of an epidural abscess, and prosthesis-related cases of osteomyelitis. Radiographs are and progression of the infection to cause spinal cord injury and often normal but may show subtle signs of and permanent paralysis. Fortunately, this occurs in less than 15% of loosening of the prosthetic components. It is difficult to distin- cases of vertebral osteomyelitis. Patients who are at increased risk guish mechanical from infectious loosening; therefore, joint for paralysis include older adults, those with cervical spine osteo- aspiration, synovial fluid analysis, and bone biopsy performed in myelitis, and those with serious underlying diseases (eg, rheuma- a sterile operative setting are undertaken to establish a firm toid arthritis, diabetes mellitus). diagnosis. Other imaging techniques such as CT and MRI are used The diagnostic procedure of choice for vertebral osteomyelitis but are difficult to interpret because of scatter from the metallic is needle biopsy, which can isolate the causative organism. Patients components and postsurgical changes. Systemic antibiotics cannot who present with a clinical picture consistent with that of vertebral penetrate the biofilm, and surgical removal of the prosthesis is osteomyelitis require rapid diagnostic confirmation through usually the only way to cure the infection. imaging or by direct needle biopsy performed by a radiologist or Puncture wounds to the feet have approximately a 2% inci- surgeon, with or without CT guidance. A delay in the definitive dence of development of osteomyelitis. The causative organism is diagnosis puts the patient at risk for the progression of vertebral usually S. aureus or beta-hemolytic streptococcus. P. aeruginosa is osteomyelitis to spinal cord compression. commonly associated with plantar wounds that occur while a Patients with vertebral osteomyelitis initially require IV anti- person is wearing rubber-soled shoes. Puncture wounds to other biotic therapy based on the offending microorganism and its parts of the body are typically nosocomial and are often due to susceptibility. Vertebral osteomyelitis can usually be successfully subclavian venipuncture, fetal scalp monitoring, and other inva- treated with antibiotics alone; surgery may be required for diag- sive procedures. nostic purposes, when there is spinal cord compression, for abscess drainage or débridement, for correction of the progressive Diabetic Foot Osteomyelitis. The pathologic changes deformity, and if the infection recurs after adequate treatment.5 induced by long-standing diabetes mellitus, such as compromised Diskitis is a variant of vertebral osteomyelitis. The disk is an vascularity, encourage the development of osteomyelitis. The avascular structure that depends on nutrient diffusion from typical patient with diabetic foot osteomyelitis is older than 50 adjacent blood vessels in the vertebral body and endplates. The years and has advanced, insulin-dependent diabetes. Over 60% of

Downloaded for dr.Rahmat Dani Satria, M.Sc, Sp.PK ([email protected]) at Universitas Gadjah Mada from ClinicalKey.com by Elsevier on July 28, 2018. For personal use only. No other uses without permission. Copyright ©2018. Elsevier Inc. All rights reserved. 1700 PART III Medicine and Surgery | SECTION Twelve Infectious Diseases

patients report polyneuropathy, over 50% report retinopathy, remarkably persistent or evolve even in the face of prolonged and at least 30% have concurrent cardiovascular disease. The antibiotic therapy; therefore, treatment commonly involves neuropathy leads to repetitive trauma and subsequent foot ulcers. surgery. Once the skin has been violated and infected, the altered host defenses of diabetic patients make it easier for infection to occur Differential Diagnosis and spread. Hyperglycemia resulting from the infection allows bacteria to proliferate, impairs leukocyte function, and results Many processes involving bone may masquerade as osteomyelitis. in defective , abnormal phagocytosis, decreased bacte- Bone tumors, such as osteoid osteomas and chrondroblastomas, ricidal function, defective antibody synthesis, and decreased may produce local pain and radiographic changes consistent with complement levels, all of which impair healing and exacerbate osteomyelitis, such as small, round, radiolucent lesions. Ewing’s osteomyelitis. The infection typically starts in the periosteum of sarcoma is a tumor of in children that can be the phalanges, spreads to the cortex, and eventually disrupts mistaken for osteomyelitis. Metastatic bone tumors and lympho- medullary bone. mas should also be considered in the differential diagnosis of Local findings in diabetic foot infections include swelling, osteomyelitis. Finally, occult fractures, such as buckle fractures in erythema, and sometimes pain. Indolent ulcers and frank cellulitis children, present with point tenderness that may be mistaken for are seen in more than 50% of cases. Because the process is often osteomyelitis. chronic, radiographic changes are often notable. Mottled lytic lesions are typical, and air may be present in the soft tissues. The Diagnostic Testing only reliable way for the bacteriologic diagnosis to be made is by surgical culture of the bone; however, wounds that can be probed Laboratory Tests all the way to the bone have a 90% positive predictive value for osteomyelitis.6 Bone biopsy for diabetic foot osteomyelitis has a Initial evaluation in the emergency department (ED) often reported sensitivity of 94%. Diabetic foot osteomyelitis is usually involves laboratory and radiographic evaluation, but the gold polymicrobial. S. aureus is the most common pathogen; other standard to confirm diagnosis is bone biopsy and culture, which organisms include streptococci, Enterobacteriaceae, and anaer- also helps guide treatment. Laboratory data are not specific and obes. Surgical treatment with amputation had been the mainstay can only suggest the diagnosis of osteomyelitis. In acute osteomy- of treatment; however, a 10-week antibiotic treatment regimen, elitis, the white blood cell (WBC) count can be elevated—typical including IV administration followed by oral antibiotics, can be values range from normal to 15,000/mm3—whereas in chronic successful.7 osteomyelitis, the WBC count is often normal. The ESR, a nonspecific measure of inflammation, is more Osteomyelitis in Sickle Cell Disease. Patients with sickle helpful than the WBC count. The ESR is a relatively sensitive cell disease (so-called sicklers) are at increased risk for hematog- marker for infection, and many series have reported elevated ESR enous infection, including osteomyelitis. In contrast to AHO in and CRP values8 in patients who have confirmed osteomyelitis. nonsicklers, AHO in children with sickle cell disease usually affects An elevated ESR in the presence of appropriate physical findings the diaphysis instead of the metaphysis. Also, although S. aureus should lead one to suspect osteomyelitis, but a normal or slightly is the most common bacterium in children with sickle cell disease elevated ESR does not eliminate the diagnosis. Other inflamma- who have osteomyelitis, Salmonella spp. are the next most common tory conditions, such as cellulitis, can cause an elevated ESR, infecting organism. Reasons for this are not completely under- although the degree of elevation of the ESR is often higher with stood, although it has been postulated that microinfarcts in the osteomyelitis. In the evaluation of a diabetic foot infection, an ESR bowel allow Salmonella bacteremia to seed the bloodstream and greater than 70 mm/hr predicts the presence of an underlying lead to hematogenous osteomyelitis. bone infection. The differentiation of bone infection from bone infarction in The CRP level, another nonspecific marker of inflammation, sickle cell patients is a challenge. Fever, toxic appearance, and increases within the first 24 hours of infection, peaks within elevated ESR are more commonly associated with osteomyelitis approximately 48 hours, and is usually normal within 1 week of than with bone infarction. Plain radiographs are not helpful in therapy. The CRP level may be a better early indicator of disease, distinguishing between the two entities, but MRI has been proving but the ESR is most valuable in following response to treatment. useful in differentiating between the two. Another approach is to Typically, the ESR falls steadily as osteomyelitis resolves and note the response to conservative therapy—bone infarctions increases should it recur. However, it is common to see elevations usually improve within 24 to 48 hours, whereas bone infection in one and not the other parameter, especially when there is worsens. Antibiotic treatment of osteomyelitis in the sickle cell development of a concurrent illness or the infection has pro- patient should include coverage against Salmonella with a third- gressed so that the ESR rises but the CRP rises and falls. In children, generation cephalosporin. an elevated ESR or CRP level is seen in all cases of osteoarticular infection; sensitivity of the use of both the ESR and CRP value is Chronic Osteomyelitis. Most chronic bone infections occur 98%, but a leukocytosis is reportedly seen in only 35% of cases.8 as a complication of posttraumatic infection, surgical procedures, or diabetic foot infections. The inflammatory response to infec- Diagnostic Imaging tion triggers bone resorption and cartilage destruction and ulti- mately leads to bone death (see Fig. 128.3). The necrotic bone acts Conventional Radiography. Conventional radiography is like a foreign material, providing an inanimate surface to which the initial modality of choice to evaluate osseous changes and, in microorganisms adhere. Clinical signs that the infection has most cases, will be the only imaging technique used to aid in the become chronic include the formation of sequestra and presence diagnosis of osteomyelitis. This is true even though radiographic of draining tracts or fistulas. Chronic infection is almost always evidence of osteomyelitis lags behind the clinical picture, and less polymicrobial and commonly involves anaerobes. Because sinus than one-third of patients have abnormalities on plain radio- tract culture is not a reliable method to predict which bacteria are graphs in the first 7 to 10 days after the onset of symptoms,. active in the underlying bone infection, direct biopsy of bone Conventional radiography is readily available, relatively inex- is the only option for the accurate diagnosis of most cases of pensive, and useful in the differentiation of infection from trauma chronic osteomyelitis. Chronically established infections can be and tumors. The characteristic findings of early osteomyelitis on

Downloaded for dr.Rahmat Dani Satria, M.Sc, Sp.PK ([email protected]) at Universitas Gadjah Mada from ClinicalKey.com by Elsevier on July 28, 2018. For personal use only. No other uses without permission. Copyright ©2018. Elsevier Inc. All rights reserved. CHAPTER 128 Bone and Joint Infections 1701

Fig. 128.6. CT scan of osteomyelitis. There is a diffuse moth-eaten

Fig. 128.5. Radiograph of right hemipelvis and proximal femur demon- appearance involving the right hemipelvis and right femoral head, with strate permeative moth-eaten changes, with resultant femoroacetabular abundant . (Courtesy Dr. Peter Evangelista, Depart- joint space narrowing consistent with osteomyelitis. (Courtesy Dr. Peter ment of Diagnostic Imaging, Rhode Island Hospital, Brown University, Evangelista, Department of Diagnostic Imaging, Rhode Island Hospital, Providence, RI.) Brown University, Providence, RI.)

the plain radiograph are lucent lytic areas of cortical bone destruc- more than 1 week for changes to be apparent. The CT scan can tion (see Fig. 128.4). However, lucency is not detected on radio- guide the surgeon in débridement and resection of infected bone graphs until approximately 50% of bone mineral is lost, which and in choosing a site for diagnostic biopsy. often takes up to 2 weeks from the onset of infection. Although these findings are often difficult to identify on plain radiographs, Magnetic Resonance Imaging. The use of bone scans and soft tissue edema, deep soft tissue swelling, distorted fascial planes, CT for the evaluation of osseus anatomy has been decreasing and altered fat interfaces may be present within 3 to 5 days from as the availability and image quality of MRI improves while its the onset of infection and can serve as a clue to osteomyelitis in cost decreases. MRI is useful to help diagnose osteomyelitis but the underlying bone. A periosteal reaction, hypertrophy or eleva- is not helpful in diagnosing septic arthritis. The anatomic resolu- tion of the periosteum, and presence of an involucrum can also tion of MRI is far superior to that of bone scans and plain be seen, especially in children, given their thinner periosteum (Fig. radiographs. MRI findings are often evident before an abnormal- 128.5). In advanced disease, the lytic lesions are surrounded by ity is detected by other modalities because of the earlier detection dense sclerotic bone, and sequestra may be noted. By 28 days from of bone marrow involvement and medullary or cortical destruc- the onset of osteomyelitis, 90% of the plain radiographs are tion, periosteal reaction, edema, soft tissue extension, joint effu- abnormal. sion, articular damage, and complications of osteomyelitis, such as abscess formation. Whereas the presence of ferromagnetic Radionuclide Bone Scanning. Radionuclide skeletal material is a contraindication to the use of MRI, most materials scintigraphy (bone scanning) is more sensitive than plain radiog- used in orthopedic surgery, such as titanium and chrome cobalt, raphy for the early diagnosis of osteomyelitis. Bone scanning is do not interfere with this imaging modality. Metal may cause especially useful in the presence of prosthetics or other hardware. distortion of the signal in the area adjacent to a joint prosthesis, Radionuclide scans can detect osteomyelitis within 48 to 72 hours but this does not exclude MRI in this group of patients. Optimal after the onset of infection. A radioactive tracer is injected into MRI images are obtained with a combination of spin echo T1- the bloodstream and given time to bind or accumulate in body and T2-weighted images, short tau inversion recovery (STIR) tissues, after which a camera is used to determine released radio- images, and fat-suppressed, T2-weighted images. Osteomyelitis activity. An image is created that is evaluated for an increase or produces a diminished intensity of the normal marrow signal decrease in expected uptake of the radionuclide. Given the radia- on T1-weighted images and a normal or increased signal on tion burden associated with this modality, however, in the past 2 T2-weighted images (Fig. 128.7). These findings, however, are not decades, there has been a movement away from skeletal scintigra- specific to osteomyelitis; the differential diagnoses for the MRI phy to MRI to diagnose osteomyelitis. findings in acute osteomyelitis are trauma, noninfectious inflam- matory and metabolic lesions, and cancer. In cases in which a Computed Tomography. CT may be also useful in the surgical procedure will be done to obtain a microbiologic diagno- diagnosis of osteomyelitis. The bony cortex is particularly well sis or is needed to treat osteomyelitis, MRI has obvious advantages seen on CT, and involucrum and sequestrum formation is easily over other modalities in detailing the anatomy for the surgeon. identified. CT is generally used to detect and define areas of pos- The administration of gadolinium as a contrast agent enhances sible infection in bones that are difficult to visualize on plain the interface between normal and abnormal marrow and helps radiographs, such as the sternum, vertebrae, pelvic bones, and distinguish devitalized bone from normally perfused bone. Gado- calcaneus. On CT scan, osteomyelitis appears as lucent areas (Fig. linium becomes localized in areas of increased vascularity and 128.6), and gas may be seen in bony abscess cavities. The limita- blood flow and also helps distinguish soft tissue infections, such tion of CT for the early diagnosis of osteomyelitis is the same as as abscesses and cellulitis from osteomyelitis. Whereas contrast that for plain radiography, in that the disease must be present for agents increase reader confidence in the diagnosis of osteomyelitis,

Downloaded for dr.Rahmat Dani Satria, M.Sc, Sp.PK ([email protected]) at Universitas Gadjah Mada from ClinicalKey.com by Elsevier on July 28, 2018. For personal use only. No other uses without permission. Copyright ©2018. Elsevier Inc. All rights reserved. 1702 PART III Medicine and Surgery | SECTION Twelve Infectious Diseases

A B

C

Fig. 128.7. MRI scan of osteomyelitis. A, Sagittal T1 image shows decreased signal within the talus, suggesting osteomyelitis. B, Axial T2 image demonstrating increased signal throughout the talus and distal fibula consistent with osteomyelitis (fat-suppressed image). C, Sagittal T1 image after the administration of gadolinium demonstrates a small focus of nonenhancing fluid just anterior to the distal fibula, sugges- tive of an abscess. (Courtesy Dr. Thomas Egglin, Department of Diagnostic Imaging, Rhode Island Hospital, Brown University, Providence, RI.)

they do not increase the sensitivity or specificity of the diagnosis other sites of infection can help identify the infecting bacteria. of osteomyelitis (Fig. 128.8). Blood cultures in patients with acute untreated osteomyelitis are positive for the offending bacteria approximately 50% of the Microbiologic Diagnosis time. In chronic osteomyelitis, blood cultures are almost always negative. The most definitive way to diagnose osteomyelitis is to obtain The emergency clinician’s diagnostic approach in suspected infected bone by needle aspiration or surgical resection. This also osteomyelitis has become simpler as radionuclide scintigraphy helps guide antimicrobial therapy. Culture of draining fistulas becomes obsolete. The algorithm in Fig. 128.2 provides a simpli- or sinus tracts is not an acceptable substitute because these fied approach to the diagnostic strategy in a patient with suspected cultured organisms often differ from those in the underlying osteomyelitis. A few key points should be considered with use of infected bone. Because osteomyelitis may be polymicrobial or due this algorithm: to unusual microorganisms, especially in immunocompromised • Radiographs lag behind the clinical picture. patients, cultures for fungal and anaerobic organisms should be • In infants and children, the amount of radiation exposure with included. imaging techniques must be considered. Particularly in cases of hematogenous osteomyelitis, cultures • If the clinical presentation strongly suggests osteomyelitis, a of blood, urine, cerebrospinal fluid, when necessary, and pus from lengthy diagnostic evaluation should not delay empirical treat-

Downloaded for dr.Rahmat Dani Satria, M.Sc, Sp.PK ([email protected]) at Universitas Gadjah Mada from ClinicalKey.com by Elsevier on July 28, 2018. For personal use only. No other uses without permission. Copyright ©2018. Elsevier Inc. All rights reserved. CHAPTER 128 Bone and Joint Infections 1703

AB

Fig. 128.8. MRI scan of osteomyelitis. A, There is diffuse, abnormal, decreased T1 signal increase. B, There is an increased STIR signal through the T12 and L1 vertebral bodies, with loss of the normal disk space and enhancement of these vertebral bodies consistent with osteomyelitis and diskitis. (Courtesy Dr. Peter Evangelista, Department of Diagnostic Imaging, Rhode Island Hospital, Brown University, Provi- dence, RI.)

ment. Culture specimens of blood, urine, and other appropri- In the case of posttraumatic osteomyelitis, appropriate initial ate sites should be obtained and antibiotic treatment started. emergency care may help prevent the disease. The proper manage- • Imaging studies other than plain radiographs are not required ment of open fractures in the field is to cut away surrounding to make the diagnosis of osteomyelitis. Advanced imaging is clothing, pour sterile saline or water over the exposed bone, and reserved for surgical planning cover the wound with moist sterile gauze bandages or a sterile sheet. Only in the case of severe vascular compromise to the distal Management limb should an open fracture site be manipulated or realigned because of the danger of introducing bacteria deeper into the Once the diagnosis of osteomyelitis is considered, the next step is wound. Because wound surface cultures in the ED setting are not to obtain culture specimens and commence treatment rapidly. reliable in predicting future pathogens in bone infections, they The goal of therapy is to contain the infection before bone necrosis need not be done as part of emergency care. occurs because cure rates fall dramatically once this happens. The first treatment priority is adequate coverage of Staphylo- Medical management with antibiotics is usually sufficient for coccus spp. with a penicillinase-resistant penicillin, such as oxacil- asymptomatic osteomyelitis that is coincidentally discovered lin or nafcillin, or first-generation cephalosporin. In patients with during the evaluation of a patient with fever, weight loss, or a penicillin allergy, vancomycin is an acceptable alternative; bacteremia, hematogenous infection caused by sensitive micro- however, cure rates with vancomycin are inferior to those with bacteria or fungi, or hematogenous vertebral osteomyelitis caused nafcillin or cefazolin. Vancomycin should be reserved for those by sensitive pathogens. patients with an actual type I penicillin allergy. Nonenterococcal For all other types of osteomyelitis, including contiguous focus streptococci are usually sensitive to antibiotics used to combat osteomyelitis, diabetic foot infections, posttraumatic osteomyeli- staphylococci. Gram-negative bacteria, including Enterobacteria- tis, and implant-related infection, definitive care is frequently ceae, E. coli, Proteus mirabilis, and Serratia marcescens, are rare surgical. In these cases, a discussion with an infectious disease or causes of osteomyelitis. Third-generation cephalosporins, amino- orthopedic surgery specialist, depending on the scenario and glycosides, imipenem-cilastatin, and ampicillin are the usual available services, is appropriate to plan or initiate surgical and choices for broad gram-negative coverage. Beyond this initial medical therapy. The ideal antibiotic for treatment of osteomyeli- broad-spectrum therapy, treatment for anaerobic bacteria, Pseu- tis should be bactericidal against the offending bacteria, such as domonas, and fungal organisms should be based on clinical beta-hemolytic streptococci and staphylococci (including MRSA), suspicion. have low toxicity, be chemically stable at the site of infection, and The increase in antimicrobial resistance highlights the need for be relatively inexpensive. The low pH of infected bone limits the new antibiotics to expand therapeutic options. Second-generation bactericidal action of some antibiotics, particularly the aminogly- fluoroquinolones, such as ciprofloxacin, and third-generation cosides. Cephalosporins and penicillins are more stable in this agents, such as levofloxacin, offer excellent bone and joint penetra- environment. In the ED, empirical broad-spectrum treatment of tion and are active against a broad spectrum of gram-positive and suspected osteomyelitis should be initiated with an awareness of gram-negative organisms. Because the blood concentrations of regional resistance patterns. Although gram-negative organisms orally and parenterally administered fluoroquinolones are similar, are uncommon pathogens, the serious consequences of inadequate transition to oral treatment protocols for osteomyelitis is pursued treatment justify the inclusion of anti–gram-negative coverage in after an initial course of parenteral antibiotics. Successful treat- the initial drug regimen. Once culture results are obtained, the ment of osteomyelitis correlates best with serum levels of the antibiotic regimen can be tailored. antibiotic, not the route of administration. Antibiotics should be

Downloaded for dr.Rahmat Dani Satria, M.Sc, Sp.PK ([email protected]) at Universitas Gadjah Mada from ClinicalKey.com by Elsevier on July 28, 2018. For personal use only. No other uses without permission. Copyright ©2018. Elsevier Inc. All rights reserved. 1704 PART III Medicine and Surgery | SECTION Twelve Infectious Diseases

dosed to ensure a serum level eight times greater than its minimum nosuppressed patients, especially those receiving corticosteroids, inhibitory concentration. Table 128.2 lists common treatment may have septic arthritis with minimal joint pain. It is important regimens for the variety of bacteria that cause osteomyelitis. The in obtaining the patient’s history to identify underlying joint standard recommendation is parenteral antibiotics for 4 to 6 disease, such as osteoarthritis, gout, rheumatoid arthritis, or joint weeks transitioned to an oral course of antibiotics. Treatment of surgery or a past medical history for chronic systemic disease, chronic osteomyelitis is a difficult surgical problem. A variety of immunodeficiency, prolonged steroid use, and/or history of injec- adjunctive therapies are have been investigated, such as instillation tion drug use. In these patients, a careful history may help dif- of antibiotic-containing beads into infected bone and hyperbaric ferentiate chronic joint pain from the acute pain associated with oxygen therapy.9 septic arthritis. On presentation, more than 80% of children and 40% of adults Disposition with septic arthritis have a fever; however, constitutional symp- toms such as weakness, malaise, anorexia, nausea, and diffuse Patients with osteomyelitis are admitted for IV antibiotic treat- myalgias are inconsistently reported. Many children who have ment and some will also need operative débridement. After septic arthritis will not use the involved limb. If the hip is infected, steady-state serum antibiotic levels have been achieved, patients the patient may present with referred pain to the thigh or knee. can receive outpatient IV or oral antibiotic therapy. On physical examination, tachycardia and hypotension may indicate a generalized septic process. In the neonate or infant, SEPTIC ARTHRITIS there may be a so-called pseudoparalysis of the affected limb. This can be mistaken for a neurologic problem; however, an isolated Principles true paralysis is far less common than septic arthritis. The inability of a child to bear weight on a lower extremity or to move any joint Septic arthritis is an orthopedic emergency, and the incidence spontaneously should be considered a sign of septic arthritis and appears to be increasing. Even with prompt recognition and should be adequately ruled out. appropriate care with antibiotics and joint decompression, septic In the older child and adult, signs may be more localized. The arthritis leads to a loss of function in 25% to 50% of patients. extremity will usually be held motionless in the position of great- In the United States, the incidence of septic arthritis in native est comfort, which is slight flexion. Palpation of the septic joint joints ranges from 2 to 10/100,000 and, in the subset of will cause exquisite pain, and any maneuver that stretches the patients with rheumatoid arthritis, the incidence jumps to 30 synovium, such as flexion and extension, will cause severe pain. to 70/100,000. The cardinal signs of inflammation—swelling, erythema, and Septic arthritis usually results from hematogenous migration warmth—are commonly found in the infected joint. Joint pain is of bacteria into a joint and is often a monoarticular process. Like 80% to 100% sensitive for septic arthritis, and tenderness is 100% osteomyelitis, septic arthritis may also result from the spread from sensitive. Periarticular processes such as bursitis, tendinitis, and a contiguous focus of infection or by direct inoculation of bacte- cellulitis may produce erythema, warmth, and tenderness, but ria. Direct inoculation can result from penetrating trauma or these processes can usually be differentiated from septic arthritis. iatrogenically as a consequence of joint aspiration or injection or Palpation of the joint line and maneuvers that stress the synovium from infected foreign material, such as a prosthesis. The synovial and joint are usually not painful in cellulitis. Periarticular pro- membrane extends beyond the epiphysis and attaches to the cesses also do not commonly produce an effusion. In general, the metaphysis in the knee, hip, and shoulder joints, causing the triad of fever (seen in 45 to 60% of cases), pain (seen in 75% of infection to spread from the metaphysis of the femur or humerus cases), and impaired range of motion suggests septic arthritis. One into the joint. This explains why septic arthritis may occur con- caveat with the physical examination is that an increasing number comitantly with osteomyelitis, with infection spreading from bone of patients have been receiving chronic immunosuppressive to joint, and osteomyelitis may also be the result of septic arthritis. drugs; in these patients, the classic history and examination find- The most commonly isolated organism is S. aureus. Polyarticular ings may be significantly less dramatic than in their immunocom- involvement is present in less than 10% of pediatric cases and petent counterparts. less than 20% of adult cases. In children, concomitant infection is seen most of the time and can be predicted by age older than 3.6 Complications years, CRP level more than 13.8 mg/L, more than 3 days of symp- toms, platelet count <314 × 10 cells/µL, and absolute neutrophil Septic arthritis leads to two types of serious complications, those count (ANC) >8.6 × 10 cells/µL. Patients with more than three involving the joint itself and those that are systemic. The introduc- of these findings would benefit from an MRI to identify any tion of bacteria into a joint triggers a profound immune response adjacent foci.10 A reactive arthritis, which is more common than that leads to destruction of the articular cartilage. Bacteria, host bacterial arthritis, is a sterile secondary inflammation of a joint, synovial cells, chondrocytes, neutrophils, and all with no identifiable infecting microorganisms in the synovial release enzymes and inflammatory chemicals such as collagenase, fluid. Commonly, reactive arthritis occurs after a systemic viral elastase, hyaluronidase, lipase, and lipoproteinase, which may be infection but can also develop after a group A streptococcal destructive to the joint. Damaged articular cartilage has limited infection. repair capacity, and a common result of articular cartilage destruction is arthritis or ankylosis, which results in a stiff immo- Clinical Features bile joint. Children are at great risk for epiphyseal damage if the infection History and Physical Examination extends through subchondral bone. This can result in impaired growth and limb length discrepancy.11 Other tissues adjacent to Septic arthritis is usually more acute in onset than osteomyelitis. the joint can be invaded, leading to suppurative destruction of The predominant symptom of septic arthritis is joint pain, exac- bursae, tendons, ligaments, and/or muscles. Sinus tracts may lead erbated with range of motion. The lower extremity is more the infection out through the skin. In the hip, the pressure and commonly affected. In infants and children, this includes the knee edema of a septic synovial effusion can occlude the blood supply, and hip; in adults, the knee is the site of septic arthritis 50% of resulting in avascular necrosis of the femoral head, especially in the time, followed by the hip (25%) and shoulder (15%). Immu- neonates.

Downloaded for dr.Rahmat Dani Satria, M.Sc, Sp.PK ([email protected]) at Universitas Gadjah Mada from ClinicalKey.com by Elsevier on July 28, 2018. For personal use only. No other uses without permission. Copyright ©2018. Elsevier Inc. All rights reserved. CHAPTER 128 Bone and Joint Infections 1705

Systemic complications from septic arthritis are less common septic arthritis of the hip. Concurrent osteomyelitis is present in but the hematogenous spread of bacteria from an infected joint 20% of infants and in almost 50% of all neonates with septic can produce sepsis, septic shock, and death. Seeding of other sites arthritis. with bacteria is also a possibility, and this can produce endocar- ditis, pneumonia, and abscesses.12-14 Gonococcal Septic Arthritis. In the United States, N. gonorrhoeae is the most common cause of septic arthritis in sexu- Clinical Subsets of Septic Arthritis ally active patients. A person with gonorrhea of the urethra, cervix, rectum, or pharynx has a 1% to 3% chance for the develop- Bites. The human mouth is a polymicrobial environment ment of disseminated gonococcal infection (DGI). More than comprised of aerobic organisms, such as Staphylococcus, oral 75% of cases of DGI occur in women, possibly because of their gram-negative rods, such as Eikenella corrodens, and anaerobes, increased risk of asymptomatic infection. DGI is common during such as Fusobacterium, making bone and joint infections caused pregnancy or after menstruation, when the alkaline vaginal envi- by human bites difficult to treat. Similarly, animal bites also lead ronment makes the organisms more resistant to host defenses in to a polymicrobial infection, with Pasteurella multocida an impor- the bloodstream and therefore more likely to disseminate. tant additional organism. Antibiotics should be empirically The classic triad of gonococcal bacteremia is migratory poly- started, but treatment also requires drainage and débridement. arthritis, tenosynovitis, and dermatitis. Asymmetric polyarthral- gia, which may be migratory, is the most common presenting Infants and Children. Septic arthritis is more common in complaint, occurring in two-thirds of cases; 25% of patients have children than in adults, and the incidence of septic arthritis is monoarthralgia. Polyarthralgia is usually asymmetric and most twice that of osteomyelitis in children. Two-thirds of pediatric frequently involves the knee, although the elbow, wrist, metacar- cases occur in children younger than 2 years, and boys are affected pophalangeal, and ankle joints are also affected. The sacroiliac and twice as often as girls. The offending agent in septic arthritis varies sternoclavicular joints may be involved, although these sites are with age. In the post–H. influenzae vaccine era, overall, S. aureus far less common. Hemorrhagic pustules on the skin, scattered, (methicillin-sensitive more than methicillin-resistant) is the most painless, nonpruritic, small (0.5- to 0.75-cm) papules distributed common infecting organism in all pediatric and adult age groups, below the neck that can involve the palms and the soles, are seen followed by group A streptococci and Streptococcus pneumoniae. in 41% of cases. These papules can turn into pustules on a broad In neonates, group B streptococci, S. aureus, and gram-negative erythematous base with a necrotic or hemorrhagic center. There enteric bacilli are usual pathogens. Candida albicans should also are usually fewer than 50 lesions, distinguishing DGI from the be considered in neonates and premature infants. K. kingae has rash of meningococcus. been emerging as an important cause of septic arthritis and Septic arthritis develops in approximately 40% of patients with osteomyelitis in children younger than 2 years. In children between DGI. It is usually a monarticular process, although polyarticular 3 months and 5 years of age, concomitant respiratory infection or septic arthritis has been reported. The patient will present with otitis media is often present. Prior trauma or skin infection may classic signs of a septic joint, including a joint effusion, warmth, be more common with staphylococcal septic arthritis. tenderness, decreased range of motion, and marked erythema. The Kocher criteria can be used to help identify children with There is usually no clear progression of DGI and polyarthralgias septic arthritis of the hip. However, the sensitivity of the algorithm to purulent monarticular arthritis, and many patients are afflicted has been challenged15 and should be used wisely. The four criteria with dermatitis and tenosynovitis without the development of are fever (temperature ≥ 38.5° C [101° F]), non–weight-bearing true arthritis. Some strains of N. gonorrhoeae that produce DGI on the affected side, ESR greater than 40 mm/hr, and peripheral favor the development of tenosynovitis and dermatitis, whereas blood WBC count more than 12,000 cells/mm3 (Table 128.3). others favor the development of purulent arthritis. Laboratory work, including and determi- The diagnosis of gonococcal arthritis is made by synovial fluid nation of the ESR and CRP level, are part of the routine evaluation culture results, but gonococci are recovered from synovial fluid in of the limping child but, individually, do not have adequate sen- less than 50% of cases. In septic arthritis due to gonorrhea, the sitivity or specificity to rule in or rule out the diagnosis. A synovial synovial fluid WBC count is often less than 50,000 cells/mm3, fluid analysis should also be done if there is any suspicion for Gram stains of aspirated joint fluid are positive for bacteria only septic arthritis. 25% of the time, and cultures of the joint fluid are negative in Even when cultures of synovial fluid and blood are tested, a approximately 50% of cases. This may be due to poor culture causative organism is not discovered in up to 30% of cases of techniques or because a suppurative reactive process can occur in septic arthritis in children. Prior antibiotic treatment in children the joint in DGI, even when bacteria are no longer present. When decreases the yield on synovial fluid cultures from 80% to 38%. gonococcal arthritis is suspected, cultures of the synovial fluid In the pediatric population, the hip and knee have equal rates of should be plated on prewarmed chocolate agar for the highest infection, with each accounting for about one- third of infections. yield, and cultures for N. gonorrhoeae should be obtained from In children, hematogenous osteomyelitis is often associated with mucosal surfaces, because these may be the only places where bacteria are readily recovered. Cultures of the genital tract, pharynx, or rectum will be positive in 80% of cases of gonococcal TABLE 128.3 arthritis. The use of urine nucleic acid amplification tests to detect gonorrhea has been gaining in popularity, given their ease of use. Kocher Criteria However, in women, vaginal swabs detect more infections than urine sample,16 whereas in men, detection of gonorrhea from NO. OF KOCHER LIKELIHOOD OF SEPTIC ARTHRITIS urine samples is equivalent to rates of detection from urethral CRITERIA MET OF THE HIP swabs.17 1 3 Gonococcal septic arthritis responds rapidly to antibiotic treatment and, unlike other types of bacterial arthritis, rarely 2 40 causes permanent damage to the joint. Patients with gonococcal 3 93 septic arthritis require hospital admission, with antibiotic cover- 4 99 age against the likely pathogens until laboratory results are avail- able. With the rise in fluoroquinolone-resistant gonorrhea, the

Downloaded for dr.Rahmat Dani Satria, M.Sc, Sp.PK ([email protected]) at Universitas Gadjah Mada from ClinicalKey.com by Elsevier on July 28, 2018. For personal use only. No other uses without permission. Copyright ©2018. Elsevier Inc. All rights reserved. 1706 PART III Medicine and Surgery | SECTION Twelve Infectious Diseases

recommended treatment of gonococcal arthritis is a third- antibiotics; adults may show an increased incidence of persistent generation cephalosporin, such as ceftriaxone, ceftizoxime, or joint swelling months to years after the initial infection, even after cefotaxime. Patients are given the first dose via the IV route or appropriate antibiotics. A small minority of patients have chronic intramuscularly in the ED and admitted until culture results are Lyme disease despite appropriate treatment, with symptoms available. If the diagnosis of gonococcal arthritis can be definitely similar to those of chronic fatigue syndrome. established in the ED, patients with reliable follow-up can be treated with an IV or intramuscular dose of antibiotics and then Periprosthetic Joint Infections. Infections occurring after sent home with an oral regimen, which should be continued for joint replacement are a challenging and dangerous complication 1 week. of arthroplasty, with rates reported to be 1% to 2% at 2 years postoperatively in native joints and up to 7% in patients undergo- Lyme Arthritis. Lyme disease, the most common tick borne ing joint revision.19,20 The prosthesis and cement are foreign disease in the United States, is caused by infection with a spiro- bodies and are ideal sites for bacterial colonization. The most chete, Borrelia burgdorferi. Transmitted by the Ixodes tick, it is an common infectious agents are S. epidermidis (40% of cases), S. important cause of arthritis in endemic areas, and its incidence aureus, methicillin-sensitive and methicillin-resistant (20%), and has been increasing. Lyme disease has been reported in all 50 streptococcal species (20%). Risk factors for periprosthetic joint states, but endemic areas, including Maryland, Massachusetts, infections have been identified to be rheumatologic disease, pre- Minnesota, New Jersey, New York, Pennsylvania, Wisconsin, Con- operative anemia, coagulopathy, diabetes, depression, and low necticut, Delaware, and Rhode Island, account for 93% of all cases socioeconomic status.20 The American Academy of Orthopaedic annually. There is a bimodal age distribution in children aged 5 Surgeons clinical practice guideline summary recommends that to 9 years and adults aged 55 to 59 years. Children infected by B. patients who present to the ED should initially be stratified to high burgdorferi are more likely than adults to have arthritis as the or low probability for a periprosthetic joint infection. As with initial manifestation of the disease. Whereas it is important to many diagnostic maneuvers performed in the ED, a pretest prob- determine a history of a tick bite, up to 30% of people do not ability helps guide the evaluation.19 remember being bitten. There are three phases of the infection— On history and physical examination, the patient will complain early localized, early disseminated, and late. of pain that is constant and present at rest, along with impaired Arthritis, which is the most distinguishing feature of late-stage function of the joint secondary to loosening of the hardware. Lyme disease, develops in up to 60% of untreated Lyme patients Radiographs may also reveal movement of the prosthesis, bone and is manifested months after disease onset. After infection, erosion, new subperiosteal bone growth widening, or more than spirochetes are disseminated and invade synovial joints, resulting a 2-mm lucency at the bone-cement interface. The investigation in a profound immune response, similar to that seen in bacterial starts with laboratory data, including ESR and CRP level. If both arthritis. Patients with Lyme arthritis present with migratory test results are negative, a periprosthetic infection is unlikely polyarthralgia that also involves bursae and tendons. This usually (negative likelihood ratio, 0–0.06); when both test results are evolves into a monarticular process and usually involves single positive, a periprosthetic infection must be considered (likelihood large joints. More than 90% of patients report knee inflammation, ratio, 4.3–12.1). However, many inflammatory processes can but other affected joints include the wrist, elbow, ankle, and hip. result in an elevation of the ESR and CRP level, and these are not The Centers for Disease Control and Prevention has defined Lyme specific tests. The use of either test alone is less reliable, and no arthritis as “recurrent, brief attacks (weeks or months) of objec- definitive conclusion can be drawn with just one result. In patients tive joint swelling in one or a few joints, sometimes followed by with an elevated ESR or CRP level in whom a prosthetic joint chronic arthritis in one or a few joints.” infection is suspected, the next step is to consult the orthopedic The rash is often overlooked by patients and is generally not service to obtain fluid from the joint under sterile conditions. present in patients when they present with arthritis. However, If synovial fluid is submitted for analysis, a synovial WBC fever is noted in up to 50% of all children who have Lyme arthri- count more than 1100 cells/mm3 with more than 64% neutrophils tis.18 Clinically, the arthritis is similar to other inflammatory is a sensitive and specific marker for periprosthetic joint infection processes of the joint and includes warmth, erythema, swelling, in a patient with an elevated ESR (>30 mm/hr) and CRP level and pain on motion of the joints; however, the effusion is usually (>10 mg/L). Aspiration may be more difficult in this situation large and out of proportion to the patient’s complaints. The effu- because of scarring and alteration of the joint space. Consultation sion also generally recurs after aspiration, even when the joint is with the patient’s orthopedic surgeon about the decision to appropriately treated. perform joint aspiration in the ED and the selection and timing The most widely used test for the diagnosis of Lyme disease is of antibiotics in suspected periprosthetic joint infection is advis- the serum antibody titer, including enzyme-linked immunosor- able. Patients who have received antibiotics within 2 weeks will bent assay (ELISA) and Western blot testing, but serum testing have a very low yield with intraarticular cultures, even if infection does not differentiate between acute and past infections. Synovial is present. Because of the difficulty in isolating infectious organ- fluid analysis is not helpful in distinguishing Lyme arthritis, but isms from the prosthetic joint, even if done intraoperatively, it usually reveals an inflammatory process with WBC counts that orthopedists have generally recommended that antibiotics not be have been reported to have a very wide range, from 500 to 98,000 started until after culture specimens are obtained.71 cells/µL. Arthrocentesis cannot differentiate between bacterial and Lyme arthritis because serologic analysis is similar. Cultures of Patients With Existing Joint Disease. Patients with synovial fluid in Lyme arthritis are usually nondiagnostic. Testing underlying joint disease, especially rheumatoid arthritis and a of synovial fluid with ELISA or Western blot methods for Lyme crystal arthropathy, are more likely to have septic arthritis than disease is not recommended because no consensus exists on how their counterparts with normal joints. If septic arthritis is sus- to interpret these data. Lyme arthritis can be successfully treated pected, laboratory and radiographic evaluation is of lower yield with oral doxycycline, amoxicillin, or cefuroxime for 30 days. If and, to reduce mortality, antibiotics are started immediately this is unsuccessful, patients can be retreated with the same oral after synovial fluid is sent for testing. In patients with a crystal regimen for another 30 days, or the antibiotic can be changed to arthropathy, neutrophil invasion secondary to septic arthritis also IV ceftriaxone for 14 to 30 days. leads to increased precipitation and release of crystals. Therefore, Fortunately, Lyme arthritis has an excellent prognosis. Up to the emergency clinician who discovers crystals on joint fluid 95% of children remain asymptomatic after a single course of aspiration should not abandon the search for an infectious agent.

Downloaded for dr.Rahmat Dani Satria, M.Sc, Sp.PK ([email protected]) at Universitas Gadjah Mada from ClinicalKey.com by Elsevier on July 28, 2018. For personal use only. No other uses without permission. Copyright ©2018. Elsevier Inc. All rights reserved. CHAPTER 128 Bone and Joint Infections 1707

Atypical Joints. Septic arthritis can be particularly difficult tender as it is with septic arthritis, and joint fluid cell counts are to diagnose and treat if it occurs in fibrocartilaginous joints, such usually below 50,000 cells/mm3. as the sternoclavicular, acromioclavicular, and sacroiliac joints and the symphysis pubis. Septic arthritis of the axial skeleton, Diagnostic Testing especially of the sternoclavicular joint, is commonly seen in injec- tion drug users, with Pseudomonas a common infecting agent. In Serum and Urine Tests patients who do not have other predisposing factors, the most common bacterial causes are S. aureus and S. epidermidis. The Blood tests are not consistently helpful in making a diagnosis presentation is usually pain and point tenderness over the involved of septic arthritis. Serum leukocytosis is nonspecific and nonsen- joint. Fever and an elevated ESR are commonly reported, although sitive for diagnosis of septic arthritis. Traditionally, a serum they are not always present because of the suppressed immune WBC count more than 10,000 cells/mm3 may suggest a systemic status of the patient. When imaging is required, CT and MRI are illness but is present in only 50% of patients with septic arthritis, preferred and are helpful in the diagnosis of septic arthritis in the and many sterile inflammatory processes create a similar leuko- fibrocartilaginous joints. cytosis. The ESR is elevated in approximately 90% of cases and, along with the CRP level, can be used to help diagnose the infec- Differential Diagnosis tion and track resolution. When low thresholds are used in the ED, the sensitivity of ESR is reported to be 98%, with a cutoff of Many disease processes can be confused with septic arthritis. 10 mm/hr or more, and the sensitivity of CRP is 92%, with a Metaphyseal osteomyelitis may mimic septic arthritis because the threshold of 20 mg/L or more.21 A sensitivity of 96% for an ESR adjacent joint may develop an effusion, and the two infections can higher than 30 mm/hr has been demonstrated. A procalcitonin be concurrent. Juvenile rheumatoid arthritis is usually more level more than 0.5 ng/mL is another possible serum marker for gradual in onset and produces polyarticular arthritis in children septic arthritis, but is also nonspecific and is often not readily younger than 16 years but may be manifested as a monoarticular available in the ED.22 process that mimics septic arthritis. Toxic or transient synovitis, Two sets of blood culture specimens should be obtained; an inflammatory process common in children, especially after however, blood cultures reveal the infecting organism in only 25% an upper respiratory infection, can be confused with septic to 50% of cases. Cultures of infectious foci, such as the throat, arthritis. It occurs in the 3-month to 6-year age range, usually cervix, and urine, may demonstrate the bacteria responsible for affects the hip, and is a self-limited disease, with no long-term septic arthritis. Urine leukocyte esterase has been studied as a morbidity. Children with transient synovitis have less pain with possible new indicator for a septic arthritis; it is especially sensitive passive joint motion than patients with septic arthritis; they and specific in prosthetic joints, with initial studies showing a high do not usually have a fever or appear ill but tend to favor the a sensitivity and specificity.23,24 unaffected leg, as in septic arthritis. The diagnostic evaluation typically reveals a normal WBC count and ESR and no radio- Joint Fluid Analysis graphic abnormalities. Other diseases of the hip in children that are included in the The diagnosis of septic arthritis requires joint fluid for culture and differential diagnoses are Legg-Calvé-Perthes disease (avascular analysis. It is fortunate that the knee joint is the most likely to be necrosis of the femoral head) and slipped capital femoral epiphy- infected and the easiest to aspirate. Aspiration of other joints, such sis; however, these processes are not as acutely disabling as septic as the hip, usually requires interventional radiology or orthopedic arthritis. Rheumatic fever commonly presents with a migrating surgical consultation. When violating the joint capsule, septic polyarthritis and may mimic gonococcal bacteremia. Patients with technique should always be practiced, even though the risk of Lyme arthritis are not as debilitated as those with septic arthritis introducing infection into a joint during intraarticular aspiration but, in endemic areas, serum antibody titers should be determined or injection has been reported to be between 1 in 2000 to 15,000 during evaluation of the patient with an effusion. injection, which seems to be related to the number of precautions In the adult, osteoarthritis, gout, and pseudogout may produce taken to avoid it. findings on joint examination similar to the findings of septic Because joint fluid analysis is not performed as often as other arthritis. Other arthropathies, such as psoriatic arthritis, arthritis diagnostic tests in the ED, a joint fluid protocol is useful to ensure associated with inflammatory bowel disease, ankylosing spondy- that all necessary tests are prepared and ordered properly. Joint litis, crystal-induced arthritis, and drug-induced arthritis should fluid cultures must be inoculated as soon as possible after the fluid also be considered in the differential diagnosis of septic arthritis. is obtained. To increase the bacterial yield from joint fluid, it is Collectively, these are known as the seronegative spondyloar- advised to inoculate blood culture bottles with joint fluid imme- thropathies. Trauma to the joint can produce synovitis and diately after joint aspiration. This allows some bacteria, which hemarthrosis, which may be mistaken for septic arthritis. In a would normally die before being inoculated on culture media in patient with hemophilia, hemarthrosis causes joint inflammation the laboratory, to survive and grow in the blood culture bottle and destruction, and there may be superimposed infection. (brain-heart infusion broth). The sample should include special Reactive arthritis has traditionally been considered to be a media to test for fastidious organisms such as N. gonorrhoeae. sterile inflammatory response to a distant infection. However, Anaerobic and fungal organisms are cultured in patients with risk antigens from the infectious trigger are often present in the joint. factors for these infections. Several viral and bacterial microorganisms can produce reactive The definitive test to determine bacterial arthritis is synovial arthritis. The most recognized syndrome is poststreptococcal culture. When only a small volume of synovial fluid is recovered reactive arthritis. Some other common organisms that cause reac- from a joint aspiration, the single most important test is a bacte- tive arthritis are Chlamydia, Salmonella, Shigella, B. burgdorferi rial culture. Culture of the synovial fluid or of synovial tissue itself, (Lyme disease), Yersinia, human T-lymphotropic virus type 1, obtained by arthrotomy, is the only definitive method for the rubella virus, hepatitis B virus, adenoviruses, parvovirus, and diagnosis of infectious arthritis. If extra fluid is available after a Epstein-Barr virus. Reactive arthritis can usually be distinguished culture specimen is obtained, other tests can be performed. from septic arthritis because it tends to involve multiple joints in Gram stain, synovial WBC count and differential, percentage a migratory pattern, the inflammatory process is less severe with of polynuclear cells, crystal analysis, and joint fluid glucose level reactive arthritis, there is less effusion, the joint is not as hot or have traditionally been used to differentiate bacterial arthritis

Downloaded for dr.Rahmat Dani Satria, M.Sc, Sp.PK ([email protected]) at Universitas Gadjah Mada from ClinicalKey.com by Elsevier on July 28, 2018. For personal use only. No other uses without permission. Copyright ©2018. Elsevier Inc. All rights reserved. 1708 PART III Medicine and Surgery | SECTION Twelve Infectious Diseases

from other joint diseases. Other tests of the synovial fluid are commonly performed, but many studies have refuted the efficacy of other additional tests. Even with an adequate joint fluid sample, proper culture techniques, and the presence of fastidious organisms, synovial Gram staining results in clinically suspected septic arthritis are negative about one-third of the time, likely due to the planktonic state of the bacteria in the joint. A positive result of Gram staining can be used to guide antibiotic treatment; however, empirical treatment should not be delayed if the result is negative because Gram staining has a 45% to 71% false-negative rate. Traditionally, a synovial fluid leukocyte count of more than 50,000 cells/mm3 with a predominance of polymorphonuclear leukocytes was used to define septic arthritis, but other processes can produce similar cell counts. Up to 30% of patients with septic arthritis have been documented to have counts well below 50,000 cells/mm3. Many studies have not supported the idea that a specific range of elevation of the synovial fluid leukocyte count can be reliably used to diagnose septic arthritis. One large study found that for a synovial fluid WBC count higher than 17,500 cells/mm3, the sensitivity was 83% and specificity was 67%. The positive likeli- hood ratio at this level was 2.5, with a negative likelihood ratio of 0.25. A synovial fluid leukocyte differential count with at least 90% neutrophils suggests septic arthritis, with a likelihood ratio of 3.4; a count of less than 90% decreases the likelihood ratio. Fig. 128.9. Ultrasound image of the right hip in an 8-year-old girl with septic arthritis. A significant joint effusion can be seen just superior to There is mounting evidence that one cannot rely solely on the the round contour of the femoral head. Joint aspiration revealed purulent synovial fluid leukocyte count to exclude or include the diagnosis fluid, with a white blood cell count of 71,000/mm3. of septic arthritis; that this value should be used with the clinical, radiographic, and laboratory findings to help guide therapy as Grams staining and culture results become available. The examination of synovial fluid under polarizing microscopy for the presence of crystals may be useful in the differentiation of anemia, lymphocytic leukemia, and femoral and inflammatory from noninflammatory joint disease but is not should be considered. helpful in identifying infection in this population because the two CT and MRI provide detailed anatomic images of the joint, often coexist. The identification of crystals should not deter the and MRI can also help determine if septic arthritis is complicated emergency clinician from continuing to search for an infectious by concurrent osteomyelitis. Skeletal scintigraphy (bone scan- cause of the joint pain. ning) has been used in the diagnosis of septic arthritis, but its use has been decreasing. The main advantage of skeletal scintigraphy Imaging is for the detection of septic arthritis earlier than with other imaging techniques. In septic arthritis, scintigraphy shows sym- Plain radiography is not an effective tool for the early evaluation metric areas of increased uptake on both sides of the joint. In a of septic arthritis but may detect surrounding osteomyelitis. In three-phase 99mTc scan, all three phases will be hot with septic most joints, the small areas of attachment of the synovial mem- arthritis. In general, skeletal scintigraphy is used only when there brane to bone are devoid of cartilage. These so-called bare areas is enough uncertainty about the diagnosis to warrant further at the margins of the joint appear as lucencies or erosions early in investigation, such as in the hip joint. In joints in which aspiration the course of septic arthritis. Bone beneath the articular cartilage is easier, skeletal scintigraphy has little role in diagnosis. may start to erode 1 to 3 weeks into the disease. Air in the joint may be a sign of infection with gas-forming organisms but may Management be the result of a previous joint aspiration. In patients with exist- ing joint disease, radiographs provide minimal assistance in the Septic arthritis is an orthopedic emergency and, once synovial diagnosis of septic arthritis. fluid is obtained, empirical antibiotics should be promptly admin- For joints that are not visualized, other than a physical exami- istered on the basis of Gram stain results, when possible, if the nation, a variety of modalities are available to help detect a joint diagnosis is strongly suspected (Table 128.4). Whereas most joint effusion, which under the right circumstances could suggest septic infections require surgical joint decompression, there are a few arthritis. However, the differential diagnoses for a joint effusion is cases in which medical management will suffice, such as with long, and aspiration of the fluid must be done to make the diag- gonococcal septic arthritis and Lyme arthritis. nosis. Ultrasonography is a useful modality to help detect a joint Unlike most other infectious emergencies encountered in the effusion and assist in joint aspiration, particularly of the hip25 (Fig. ED, when time to antibiotic administration decreases morbidity 128.9). CT and MRI can identify joint fluid but not necessarily an and mortality, definitive management for most cases of septic effusion, because a volumetric analysis cannot be done to assess arthritis requires surgical intervention and a prolonged course of the amount of fluid. In adult patients with an antalgic gait and antibiotics. Therefore, it is more important to obtain synovial painful internal and external rotation of the hip, MRI findings of fluid for Gram staining and culture than to start antibiotics, bone marrow edema led to a diagnosis of septic arthritis only 6% because this will guide long-term antibiotic treatment. In the of the time. Other conditions that cause bone marrow edema hemodynamically stable patient in whom septic arthritis is a include reactive arthritis, transient , avascular necro- strong consideration, antibiotics should be held until blood and sis, osteoarthritis, tuberculous arthritis, osteomyelitis, sickle cell synovial fluid cultures are obtained.

Downloaded for dr.Rahmat Dani Satria, M.Sc, Sp.PK ([email protected]) at Universitas Gadjah Mada from ClinicalKey.com by Elsevier on July 28, 2018. For personal use only. No other uses without permission. Copyright ©2018. Elsevier Inc. All rights reserved. CHAPTER 128 Bone and Joint Infections 1709

TABLE 128.4 Guidelines for Choice of Empirical Antibiotic Based on Gram Staining Results GRAM STAIN OR CLINICAL CONDITION PROBABLE ORGANISM PREFERRED ANTIBIOTICS ALTERNATIVE ANTIBIOTICS Gram-positive cocci Staphylococcus aureus Nafcillin or cefazolin Clindamycin Streptococci Trimethoprim-sulfamethoxazole Vancomycin Gram-negative cocci or negative stain Neisseria gonorrhoeae Ceftriaxone Doxycycline Healthy, sexually active patient Gram-negative bacilli Pseudomonas aeruginosa Piperacillin ± gentamicin Third-generation cephalosporin Enterobacteriaceae Gram-positive bacilli Propionibacterium acnes Penicillin G Nafcillin Vancomycin Reprinted with permission from Rao N, Esterhai JL Jr: Septic arthritis. In Cierny G 3rd, McLaren AC, Wongworawat MD, editors: Orthopaedic knowledge update: musculoskeletal infection. Rosemont, IL, 2009, American Academy of Orthopaedic Surgeons.

The selection of antibiotics for the treatment of septic arthritis a delay in diagnosis and treatment, patients with underlying joint is outlined in Table 128.2. In most cases, the emergency clinician disease (especially rheumatoid arthritis), those with polyarticular does not know the identity of the causative organism, but treat- septic arthritis, and those who have positive blood cultures. ment should be tailored to the most likely causative agents based Despite many advances in diagnosis and treatment, the overall on the patient’s age, history, and immune status. S. aureus accounts morbidity for patients with septic arthritis has not decreased in for 44% of cases and remains the predominant pathogen for all the last 3 decades. A general rule is that if the diagnosis of septic age groups. Unless gonococcal arthritis is confirmed, the antibiotic arthritis is made and treatment is initiated within 1 week of the selected should be bactericidal against S. aureus. Empirical anti- onset of symptoms, the outcome is almost always favorable. biotics active against MRSA should be considered but should be Diagnosis and rapid treatment of septic arthritis have proven to based on the prevalence of this pathogen in the community. Risk be most elusive in two groups of patients, infants and people with factors for MRSA include older age, chronic illness, and health existing joint disease. In infants and children, early symptoms can care exposure. Group B streptococci have emerged as invasive be nonspecific and difficult to assess; consequently, children with pathogens in older adults, especially those with diabetes mellitus, septic arthritis, especially of the hip, who experience a delay cirrhosis, and neurologic disease. Gonococcal septic arthritis is the in diagnosis and treatment have a disappointingly high rate most common cause of arthritis in young adults. Penicillin- and of complications. In patients with existing joint disease, septic fluoroquinolone-resistant strains have become more prevalent, arthritis may be mistaken for an acute exacerbation of the under- and a third-generation cephalosporin is the best choice for gono- lying disease process, so emergency clinicians must remain vigilant coccal arthritis. In older adults, gram-negative septic arthritis is in their pursuit of the correct diagnosis. more common, and agents such as the third-generation cephalo- sporins and aminoglycosides are added to the antistaphylococcal regimen. Establishment of good bactericidal serum levels of KEY CONCEPTS antibiotics will ensure that the levels in joint fluid are also bacte- • Skeletal infection should be considered in the differential diagnosis ricidal. In pediatrics, there is some evidence to support the early of all patients who present with bone or joint pain. administration of dexamethasone (0.15 mg/kg qid for 4 days) to 26 • Hematologic evaluation is of little value in the diagnosis of bone accelerate clinical improvement. and joint infections, with the exception of the ESR and CRP level, which are elevated in approximately 90% of cases of bone and Disposition joint infections. • The diagnostic evaluation for septic arthritis includes complete Any patient thought to have septic arthritis requires joint aspira- blood count, ESR, and CRP level. Joint aspiration is the definitive tion. Patients for whom septic arthritis is considered to be the diagnostic procedure, and synovial culture is the only reliable joint diagnosis should be given an initial parenteral dose of antibiotics fluid test for establishing a diagnosis. in the ED and admitted for culture results and continued manage- • The diagnosis of osteomyelitis involves an operative culture of the ment. If the joint fluid aspirate is not consistent with septic infected bone. MRI has become the best diagnostic modality to arthritis and clinical findings are equivocal, the patient can be detect osteomyelitis. discharged and reevaluated in 24 hours. In immunosuppressed • With suspected septic arthritis, joint fluid and blood culture patients, patients with preexisting joint disease, and patients with specimens are obtained before IV antibiotics are administered. With a joint replacement, septic arthritis can be difficult to detect. A suspected osteomyelitis, blood culture specimens are obtained, and conservative approach with in-hospital observation and treatment IV antibiotics are administered while plans are made for further should be considered if there is any possibility of septic arthritis imaging studies or surgical aspiration or resection of bone. in these patients. • The most important aspect of antibiotic treatment of suspected The prognosis for the patient with septic arthritis is favorable bone and joint infections is to provide potent bactericidal activity against S. aureus with additional empirical antibiotic coverage in most cases. From 50% to 75% of afflicted patients can expect aimed at suspected organisms on the basis of age, risk factors, and to recover completely and achieve full painless range of motion regional variability. of the afflicted joint. About one-third of patients have complica- tions such as decreased mobility or ankylosis, pain on joint movement, chronic infection, or overwhelming sepsis and death. The references for this chapter can be found online by accessing the The patients most likely to do poorly include those who have had accompanying Expert Consult website.

Downloaded for dr.Rahmat Dani Satria, M.Sc, Sp.PK ([email protected]) at Universitas Gadjah Mada from ClinicalKey.com by Elsevier on July 28, 2018. For personal use only. No other uses without permission. Copyright ©2018. Elsevier Inc. All rights reserved. CHAPTER 128 Bone and Joint Infections 1709.e1

REFERENCES

1. Cunning J, Joll C: Aids to surgery, ed 4, New York, 1919, William Wood. 15. Sultan J, Hughes P: Septic arthritis or transient synovitis of the hip in children: the 2. Conrad D: Acute hematogenous osteomyelitis. Pediatr Rev 31:464–471, 2010. value of clinical prediction algorithms. J Bone Joint Surg Br 92:1289–1293, 2010. 3. Kapoor M, et al: Role of proinflammatory in the pathophysiology of 16. Chernesky M, et al: Head-to-head comparison of second-generation nucleic acid osteoarthritis. Nat Rev Rheumatol 7:33–42, 2011. amplification tests for detection of Chlamydia trachomatis and Neisseria gonor- 4. Ceroni D, et al: Kingella kingae osteoarticular infections in young children: clinical rhoeae on urine samples from female subjects and self-collected vaginal swabs. J Clin features and contribution of a new specific real-time PCR assay to the diagnosis. Microbiol 52:2305–2310, 2014. J Pediatr Orthop 30:301–304, 2010. 17. Taylor SN, et al: Evaluation of the Roche cobas® CT/NG test for detection of 5. Zimmerili W: Vertebral osteomyelitis. N Engl J Med 362:1022–1029, 2010. Chlamydia trachomatis and Neisseria gonorrhoeae in male urine. Sex Transm Dis 6. Markanday A: Diagnosing diabetic foot osteomyelitis: narrative review and a sug- 39:543–549, 2012. gested 2-step score-based diagnostic pathway for clinicians. Open Forum Infect Dis 18. Milewski M, et al: Lyme arthritis in children presenting with joint effusions. J Bone 1:ofu060, 2014. Joint Surg Am 93:252–260, 2011. 7. Game FL: Osteomyelitis in the diabetic foot: diagnosis and management. Med Clin 19. Della Valle C, et al: Diagnosis of periprosthetic joint infections of the hip and knee. North Am 97:947–956, 2013. J Am Acad Orthop Surg 18:760–769, 2010. 8. Paakkonen M, et al: Sensitivity of erythrocyte sedimentation rate and C-reactive 20. Kurtz S, et al: Prosthetic joint infection risk after TKA in the Medicare population. protein in childhood bone and joint infections. Clin Orthop Relat Res 468:861–866, Clin Orthop Relat Res 468:52–56, 2010. 2010. 21. Hariharan P, Kabrhel C: Sensitivity of erythrocyte sedimentation rate and C-reactive 9. Shields R, et al: Hyperbaric oxygen therapy for chronic refractory osteomyelitis of protein for the exclusion of septic arthritis in emergency department patients. the sternum. Ann Thoac Surg 89:1661–1663, 2010. J Emerg Med 40:428–431, 2011. 10. Rosenfeld S, et al: Predicting the presence of adjacent infections in septic arthritis in 22. Paosong S, et al: Serum procalcitonin as a diagnostic aid in patients with acute bacte- children. J Pediatr Orthop 36:70–74, 2016. rial septic arthritis. Int J Rheum Dis 18:352–359, 2015. 11. Li X, et al: Evaluation of the modified Albee arthroplasty for femoral head loss sec- 23. Colvin OC, et al: Leukocyte esterase analysis in the diagnosis of joint infection: ondary to septic arthritis in young children: surgical technique. J Bone Joint Surg Am can we make a diagnosis using a simple urine dipstick? Skeletal Radiol 44:673–677, 93(Suppl 1):54–61, 2011. 2015. 12. Scillia A, et al: Primary osteomyelitis of the acetabulum resulting in septic arthritis 24. Tischler EH, et al: Leukocyte esterase strip test: matched for musculoskeletal infection of the hip and obturator internus abscess diagnosed as acute appendicitis. J Pediatr society criteria. J Bone Joint Surg Am 96:1917–1920, 2014. Surg 45:1707–1710, 2010. 25. Laine JC, et al: The use of ultrasound in the management of septic arthritis of the 13. Wang E, et al: Psoas abscess with associated septic arthritis of the hip in infants. hip. J Pediatr Orthop B 24:95–98, 2015. J Pediatr Surg 45:2440–2443, 2010. 26. Harel L, et al: Dexamethasone therapy for septic arthritis in children: results of a 14. Molloy A, et al: The complications of septic arthritis in the elderly. Aging Clin Exp randomized double-blind placebo-controlled study. J Pediatr Orthop 31:211–215, Res 22:270–273, 2010. 2011.

CHAPTER 128: QUESTIONS & ANSWERS 128.1. Which of the following statements regarding 128.3. Which of the following statements regarding septic osteomyelitis is true? arthritis is true? A. Contiguous focus osteomyelitis is most common in A. Despite vaccination, Haemophilus influenzae remains the knee. a frequent pathogen in children. B. Hematogenous osteomyelitis begins in medullary B. Pseudomonas aeruginosa is associated with IVDU- bone. related osteomyelitis. C. Head and neck osteomyelitis is usually from C. Pseudomonas aeruginosa is not associated with hematogenous spread. prosthetic device joint infection. D. Septic arthritis begins in joint fluid and spreads to D. The most common organism in neonates is synovium. Staphylococcus aureus. E. Septic arthritis morbidity is related to joint size. E. The most common organism in patients younger than 30 years is S. aureus. Answer: B. Hematogenous osteomyelitis begins in medullary bone and spreads outward. Contiguous focus osteomyelitis, most Answer: B. Pseudomonas is associated with IVDU-related cervical common in the foot and hand, begins outside of bone and spreads osteomyelitis and lumbar osteomyelitis in cases of prolonged inward via Volkmann’s canal. Head and neck osteomyelitis is urinary catheterization. H. influenzae has largely disappeared as a usually from a contiguous sinus or otic process. Septic arthritis joint pathogen in vaccinated children. The most common neona- begins in synovium and later involves joint fluid. Morbidity is tal joint pathogens are group B streptococci, Escherichia coli, and related to the degree of hyaline cartilage destruction. Staphylococcus epidermidis. The most common cause of septic arthritis in people younger than 30 years is gonococcal. 128.2. Which of the following associations between osteomyelitis and pathogenic organism is true? 128.4. Which of the following statements regarding acute A. Dog bite—Pasteurella hematogenous osteomyelitis in children is true? B. Freshwater wounds—Pseudomonas A. Blood cultures are not usually positive. C. Human bite—Aeromonas B. Skeletal scintigraphy is indicated in neonates. D. Intravenous drug use—Fusobacterium C. The child usually appears toxic. E. Sickle cell anemia—Haemophilus influenzae D. The most common site is the long bone epiphysis. E. There is a female preponderance. Answer: A. The following are correct associations: Freshwater—Aeromonas hydrophila Answer: C. Children may be ill but not usually toxic. There is a Intravenous drug use (IVDU)—Staphylococcus aureus 2 : 1 or 3 : 1 male predominance, with the most likely site being the Sickle cell—Pseudomonas and Salmonella distal metaphysis. Blood cultures are positive in 60% of cases. Cat or dog bite—Pasteurella multocida Scintigraphy is not useful in neonates due to a limited inflamma- Human bite—mixed with Fusobacterium, Eikenella, and tory response. Radiographs are more useful and sensitive early on Streptococcus anginosus than in adults.

Downloaded for dr.Rahmat Dani Satria, M.Sc, Sp.PK ([email protected]) at Universitas Gadjah Mada from ClinicalKey.com by Elsevier on July 28, 2018. For personal use only. No other uses without permission. Copyright ©2018. Elsevier Inc. All rights reserved. 1709.e2 PART III Medicine and Surgery | SECTION Twelve Infectious Diseases

128.5. Which of the following statements regarding vertebral E. The erythrocyte sedimentation rate (ESR) is elevated osteomyelitis is true? in sickle pain crises. A. Children are less prone to isolated diskitis. Answer: A. Sickle cell osteomyelitis is more typically seen in the B. Of the cases of epidural abscesses, 30% are due to diaphysis than in non–sickle cell situations, which more often osteomyelitis. involve the metaphysis. Fever, toxicity, and an elevated ESR suggest C. The diagnostic procedure of choice is a magnetic infection. The most likely infectious cause is still Staphylococcus resonance imaging scan. aureus, followed by Salmonella. Often, observation and response D. The most common location is the lumbar spine. to therapy (eg, analgesics, hydration) ultimately help differentiate E. Vertebral osteomyelitis typically involves a single the two. Plain technetium lights up infection and infarction. . Indium or gallium is necessary to show a hot spot of infection; an Answer: D. The incidence of associated epidural abscess is 15%. infarctive site would be cold. The most common location of vertebral osteomyelitis is the thoracic, lumbar, and cervical spine. The diagnostic procedure of 128.7. Which test will most likely confirm a diagnosis of choice is needle biopsy. The disease usually involves two vertebrae gonococcal arthritis in a female? and the disk in between. Children are more prone to isolated A. Pelvic culture diskitis, although it may also occur in adults. B. Serum gonococcal culture C. Synovial gonococcal culture done on chocolate agar 128.6. An 18-year-old woman with known sickle cell disease D. Thorough history presents with leg pain of 2 days’ duration. Her typical Answer: A. It is recommended that cervical culture be used to pain syndrome is lower extremity tibial and femur pain. confirm the likely presence of GC arthritis. Today’s episode is primarily right tibial. She complains bitterly of pain, but there are no gross findings other 128.8. When should antibiotics be administered in cases of than trace bilateral anterior tibial swelling, with no suspected septic arthritis? discernible warmth or erythema. Vital signs are A. After radiographic confirmation of the diagnosis remarkable for a low-grade fever and heart rate of 110 B. After serologic confirmation of the diagnosis beats/min. Which of the following statements regarding C. After synovial fluid is sent for culture this patient’s condition is true? D. After synovial Gram staining comes back positive A. Bony infection would be expected in the bony diaphysis. Answer: C. To guide effective antibiotic therapy, it is recom- B. Plain radiography can differentiate bony infarction mended to withhold antibiotics until synovial fluid has been from infection. obtained. C. Salmonella would be the most likely infectious cause. D. Technetium scintigraphy (bone scanning) will differentiate infection from infarction.

Downloaded for dr.Rahmat Dani Satria, M.Sc, Sp.PK ([email protected]) at Universitas Gadjah Mada from ClinicalKey.com by Elsevier on July 28, 2018. For personal use only. No other uses without permission. Copyright ©2018. Elsevier Inc. All rights reserved.